Last visit was: 18 Nov 2025, 23:45 It is currently 18 Nov 2025, 23:45
Close
GMAT Club Daily Prep
Thank you for using the timer - this advanced tool can estimate your performance and suggest more practice questions. We have subscribed you to Daily Prep Questions via email.

Customized
for You

we will pick new questions that match your level based on your Timer History

Track
Your Progress

every week, we’ll send you an estimated GMAT score based on your performance

Practice
Pays

we will pick new questions that match your level based on your Timer History
Not interested in getting valuable practice questions and articles delivered to your email? No problem, unsubscribe here.
Close
Request Expert Reply
Confirm Cancel
555-605 Level|   Weaken|                              
User avatar
AbdurRakib
Joined: 11 May 2014
Last visit: 08 Nov 2025
Posts: 465
Own Kudos:
Given Kudos: 220
Status:I don't stop when I'm Tired,I stop when I'm done
Location: Bangladesh
Concentration: Finance, Leadership
GPA: 2.81
WE:Business Development (Real Estate)
Posts: 465
Kudos: 42,843
 [270]
38
Kudos
Add Kudos
232
Bookmarks
Bookmark this Post
Most Helpful Reply
User avatar
GMATNinja
User avatar
GMAT Club Verbal Expert
Joined: 13 Aug 2009
Last visit: 18 Nov 2025
Posts: 7,445
Own Kudos:
69,780
 [37]
Given Kudos: 2,060
Status: GMAT/GRE/LSAT tutors
Location: United States (CO)
GMAT 1: 780 Q51 V46
GMAT 2: 800 Q51 V51
GRE 1: Q170 V170
GRE 2: Q170 V170
Products:
Expert
Expert reply
GMAT 2: 800 Q51 V51
GRE 1: Q170 V170
GRE 2: Q170 V170
Posts: 7,445
Kudos: 69,780
 [37]
29
Kudos
Add Kudos
8
Bookmarks
Bookmark this Post
User avatar
Divyadisha
User avatar
Current Student
Joined: 18 Oct 2014
Last visit: 01 Jun 2018
Posts: 663
Own Kudos:
1,928
 [31]
Given Kudos: 69
Location: United States
GMAT 1: 660 Q49 V31
GPA: 3.98
GMAT 1: 660 Q49 V31
Posts: 663
Kudos: 1,928
 [31]
26
Kudos
Add Kudos
4
Bookmarks
Bookmark this Post
General Discussion
User avatar
Kurtosis
User avatar
Current Student
Joined: 13 Apr 2015
Last visit: 10 Nov 2021
Posts: 1,395
Own Kudos:
5,121
 [6]
Given Kudos: 1,228
Location: India
Products:
Posts: 1,395
Kudos: 5,121
 [6]
3
Kudos
Add Kudos
3
Bookmarks
Bookmark this Post
Younger people eat in fastfood. Average age of current population is increasing - Number of fastfood restaurants will decrease

Possible weakener: As the current population ages, younger people are added to the population

A) Fast-food restaurants in Canatria are getting bigger, so each one can serve more customers. - Incorrect - Out of context. We are not bothered about the increase in capacity of fast food restaurants.

B) Some older people eat at fast-food restaurants more frequently than the average young person. - Incorrect - Irrelevant

C) Many people who rarely eat in fast-food restaurants nevertheless eat regularly in restaurants. - Incorrect - Strengthens

D) The overall population of Canatria is growing steadily. - Correct

E) As the population of Canatria gets older, more people are eating at home. - Incorrect - Supports the argument

Answer: D
User avatar
ynk
Joined: 18 Aug 2013
Last visit: 02 Nov 2017
Posts: 106
Own Kudos:
131
 [11]
Given Kudos: 127
Location: India
Concentration: Operations, Entrepreneurship
GMAT 1: 640 Q48 V28
GPA: 3.92
WE:Operations (Transportation)
GMAT 1: 640 Q48 V28
Posts: 106
Kudos: 131
 [11]
10
Kudos
Add Kudos
Bookmarks
Bookmark this Post
AbdurRakib
Fast-food restaurants make up 45 percent of all restaurants in Cantaria. Customers at these restaurants tend to be young; in fact, studies have shown that the older people get, the less likely they are to eat in fast-food restaurants. Since the average age of the Canatrian population is gradually rising and will continue to do so, the number of fast-food restaurants is likely to decrease.

Which of the following, if true, most seriously weakens the argument?

A) Fast-food restaurants in Canatria are getting bigger, so each one can serve more customers.
B) Some older people eat at fast-food restaurants more frequently than the average young person.
C) Many people who rarely eat in fast-food restaurants nevertheless eat regularly in restaurants.
D) The overall population of Canatria is growing steadily.
E) As the population of Canatria gets older, more people are eating at home.


OG 2017 New Question


A) Fast-food restaurants in Canatria are getting bigger, so each one can serve more customers.--No mention of no. of customers a restaurant can serve.Eliminate
B) Some older people eat at fast-food restaurants more frequently than the average young person.--Some people cannot help to weaken the argument.Eliminate.
C) Many people who rarely eat in fast-food restaurants nevertheless eat regularly in restaurants.--Restaurants are out of scope.only fast food restaurants is under consideration.Eliminate
D) The overall population of Canatria is growing steadily. --Ok.If population is growing that means more births to new child.i.e more youths in near future.Perfect
E) As the population of Canatria gets older, more people are eating at home.--This actually strengthens the argument.Eliminate
User avatar
RichaChampion
Joined: 28 Sep 2013
Last visit: 02 Jan 2018
Posts: 70
Own Kudos:
88
 [10]
Given Kudos: 82
GMAT 1: 740 Q51 V39
GMAT 1: 740 Q51 V39
Posts: 70
Kudos: 88
 [10]
5
Kudos
Add Kudos
5
Bookmarks
Bookmark this Post
Number & Percentages are two different things.
Percentages are actually numbers expressed on the scale of 100.

In various problems, they use number and percentages interchangeably to create a RIGHT or WRONG solution.

20% 100 < 5%1000 [20 < 50]

Fast-food restaurants make up 45 percent of all restaurants in Cantaria. Customers at these restaurants tend to be young; in fact, studies have shown that the older people get, the less likely they are to eat in fast-food restaurants. Since the average age of the Canatrian population is gradually rising and will continue to do so, the number of fast-food restaurants is likely to decrease.

The argument is saying that since the average age is gradually rising that means that the %age of older people is increasing and the percentage of young people is decreasing from 45%. Therefore the number of fast foods restaurant is likely to decrease.

Which of the following, if true, most seriously weakens the argument?

The existence or Presence or Survival of Fast Food restaurants depends on upon a minimum number of people who belong to the young age, but as the argument says that this %age is decreasing, therefore a lesser number of Young people.

CURRENT STATUS = 45% OF CURRENT TOTAL POPULATION = CERTAIN CURRENT NUMBER

But, The percentage in red is decreasing? Then how can we maintain the same number of young people?

As we discussed above with sample space - 20% 100 < 5%1000

(45-X)% OF FUTURE TOTAL POPULATION ≥ 45% OF CURRENT TOTAL POPULATION

Where,
FUTURE TOTAL POPULATION > CURRENT TOTAL POPULATION - This is what option D says.
avatar
MackayMcCoy
Joined: 24 Jan 2017
Last visit: 25 Jul 2021
Posts: 1
Given Kudos: 36
Posts: 1
Kudos: 0
Kudos
Add Kudos
Bookmarks
Bookmark this Post
Dear Experts, please assist me here: Isn't Option B valid? Although it states that "Some" older people eat more frequently than the younger people, the statement still attacks the conclusion made by the author! Although option B says "Some", the option is still strong enough to arrest the decrease of the fast food restaurants
avatar
scarlatti
Joined: 02 Nov 2017
Last visit: 28 Jan 2018
Posts: 1
Own Kudos:
1
 [1]
Given Kudos: 8
Concentration: Finance, Entrepreneurship
GMAT 1: 600 Q45 V27
GMAT 1: 600 Q45 V27
Posts: 1
Kudos: 1
 [1]
1
Kudos
Add Kudos
Bookmarks
Bookmark this Post
Vyshak
Younger people eat in fastfood. Average age of current population is increasing - Number of fastfood restaurants will decrease

Possible weakener: As the current population ages, younger people are added to the population

A) Fast-food restaurants in Canatria are getting bigger, so each one can serve more customers. - Incorrect - Out of context. We are not bothered about the increase in capacity of fast food restaurants.

B) Some older people eat at fast-food restaurants more frequently than the average young person. - Incorrect - Irrelevant

C) Many people who rarely eat in fast-food restaurants nevertheless eat regularly in restaurants. - Incorrect - Strengthens

D) The overall population of Canatria is growing steadily. - Correct

E) As the population of Canatria gets older, more people are eating at home. - Incorrect - Supports the argument

Answer: D


I have a problem with D. Just because Canatria´s population increased, it doesn´t mean that the number of young people in Canatria will increase, since that same increase in the population could have been due to immigration for example, and we can´t say that immigrants are young.
User avatar
GMATNinja
User avatar
GMAT Club Verbal Expert
Joined: 13 Aug 2009
Last visit: 18 Nov 2025
Posts: 7,445
Own Kudos:
69,780
 [14]
Given Kudos: 2,060
Status: GMAT/GRE/LSAT tutors
Location: United States (CO)
GMAT 1: 780 Q51 V46
GMAT 2: 800 Q51 V51
GRE 1: Q170 V170
GRE 2: Q170 V170
Products:
Expert
Expert reply
GMAT 2: 800 Q51 V51
GRE 1: Q170 V170
GRE 2: Q170 V170
Posts: 7,445
Kudos: 69,780
 [14]
11
Kudos
Add Kudos
3
Bookmarks
Bookmark this Post
scarlatti
Vyshak
Younger people eat in fastfood. Average age of current population is increasing - Number of fastfood restaurants will decrease

Possible weakener: As the current population ages, younger people are added to the population

A) Fast-food restaurants in Canatria are getting bigger, so each one can serve more customers. - Incorrect - Out of context. We are not bothered about the increase in capacity of fast food restaurants.

B) Some older people eat at fast-food restaurants more frequently than the average young person. - Incorrect - Irrelevant

C) Many people who rarely eat in fast-food restaurants nevertheless eat regularly in restaurants. - Incorrect - Strengthens

D) The overall population of Canatria is growing steadily. - Correct

E) As the population of Canatria gets older, more people are eating at home. - Incorrect - Supports the argument

Answer: D

I have a problem with D. Just because Canatria´s population increased, it doesn´t mean that the number of young people in Canatria will increase, since that same increase in the population could have been due to immigration for example, and we can´t say that immigrants are young.
Vyshak, it's true that choice (D) might not weaken the argument. We can think of scenarios, like the one you described, in which choice (D) would have no effect. But remember that we are looking for an answer choice that most seriously weakens the argument, not an answer choice that definitely weakens the argument.

The author says that the number of fast-food restaurants is likely to decrease because the average age is increasing. Thus, according to the author, the number of young people will probably decrease. Choice (D) seriously undermines this reasoning. If the overall population is growing steadily ("steadily" is a key word here), then that will probably counteract the effect of the aging.

Sure, we might have a steady influx of older immigrants, but if we are talking about steady growth of the overall population, it is more likely that we will see increases across all ages. Choice (D) does not disprove the author's logic with 100% certainty, but it does seriously weaken the argument.

I hope that helps!
avatar
bpdulog
Joined: 14 Aug 2012
Last visit: 19 Aug 2020
Posts: 51
Own Kudos:
Given Kudos: 221
Location: United States
GMAT 1: 620 Q43 V33
GMAT 2: 690 Q47 V38
Kudos
Add Kudos
Bookmarks
Bookmark this Post
GMATNinja
scarlatti
Vyshak
Younger people eat in fastfood. Average age of current population is increasing - Number of fastfood restaurants will decrease

Possible weakener: As the current population ages, younger people are added to the population

A) Fast-food restaurants in Canatria are getting bigger, so each one can serve more customers. - Incorrect - Out of context. We are not bothered about the increase in capacity of fast food restaurants.

B) Some older people eat at fast-food restaurants more frequently than the average young person. - Incorrect - Irrelevant

C) Many people who rarely eat in fast-food restaurants nevertheless eat regularly in restaurants. - Incorrect - Strengthens

D) The overall population of Canatria is growing steadily. - Correct

E) As the population of Canatria gets older, more people are eating at home. - Incorrect - Supports the argument

Answer: D

I have a problem with D. Just because Canatria´s population increased, it doesn´t mean that the number of young people in Canatria will increase, since that same increase in the population could have been due to immigration for example, and we can´t say that immigrants are young.
Vyshak, it's true that choice (D) might not weaken the argument. We can think of scenarios, like the one you described, in which choice (D) would have no effect. But remember that we are looking for an answer choice that most seriously weakens the argument, not an answer choice that definitely weakens the argument.

The author says that the number of fast-food restaurants is likely to decrease because the average age is increasing. Thus, according to the author, the number of young people will probably decrease. Choice (D) seriously undermines this reasoning. If the overall population is growing steadily ("steadily" is a key word here), then that will probably counteract the effect of the aging.

Sure, we might have a steady influx of older immigrants, but if we are talking about steady growth of the overall population, it is more likely that we will see increases across all ages. Choice (D) does not disprove the author's logic with 100% certainty, but it does seriously weaken the argument.

I hope that helps!

So does overall in this context mean all age groups? D is kind of vague, I thought B was the better answer
User avatar
GMATNinja
User avatar
GMAT Club Verbal Expert
Joined: 13 Aug 2009
Last visit: 18 Nov 2025
Posts: 7,445
Own Kudos:
69,780
 [3]
Given Kudos: 2,060
Status: GMAT/GRE/LSAT tutors
Location: United States (CO)
GMAT 1: 780 Q51 V46
GMAT 2: 800 Q51 V51
GRE 1: Q170 V170
GRE 2: Q170 V170
Products:
Expert
Expert reply
GMAT 2: 800 Q51 V51
GRE 1: Q170 V170
GRE 2: Q170 V170
Posts: 7,445
Kudos: 69,780
 [3]
1
Kudos
Add Kudos
1
Bookmarks
Bookmark this Post
bpdulog
So does overall in this context mean all age groups? D is kind of vague, I thought B was the better answer
As described in my last post, (D) is a bit "vague". Specifically, we cannot say with 100% certainty that (D) would weaken the argument.

The "overall" population refers to the total population. Sure, there might be a steady influx of older immigrants, causing a steady increase in overall population. But it is likely that steady growth of the overall population implies growth across all age groups.

As for choice (B), check out this explanation.

I hope that helps!
User avatar
warrior1991
Joined: 03 Mar 2017
Last visit: 03 Feb 2022
Posts: 573
Own Kudos:
437
 [3]
Given Kudos: 596
Location: India
Concentration: Operations, Technology
Products:
Posts: 573
Kudos: 437
 [3]
1
Kudos
Add Kudos
2
Bookmarks
Bookmark this Post
Argument Analysis :-

1. The argument says that Fast-food restaurants make up 45 percent of all restaurants in Cantaria.
2. Youth is a main consumer at these restaurants.
3. The average age is rising(people are moving towards old age).
4. Therefore number of restaurants are likely to decrease.

Which of the following, if true, most seriously weakens the argument?

(A) Fast-food restaurants in Canatria are getting bigger, so each one can serve more customers.
Does not matter if they become bigger. The problem will still not be solved, if the average age is rising. Irrelevant.

(B) Some older people eat at fast-food restaurants more frequently than the average young person.
This is against the premise given in the question that people tend to be young at Fast food restaurants. We cannot challenge the question stimuli. Irrelevant.

(C) Many people who rarely eat in fast-food restaurants nevertheless eat regularly in restaurants.
So what!! We are talking about how to tackle average age rising scenario. Irrelevant.

(D) The overall population of Canatria is growing steadily.
Sounds good. The overall population is growing steadily means Canatria is having a influx of newborn.They will become young one day. Hence will eat at these restaurants.
Weakens the conclusion. :)


(E) As the population of Canatria gets older, more people are eating at home.
This will strengthen the argument. If this is the case, restaurants are likely to decrease.
User avatar
saby1410
Joined: 06 Feb 2017
Last visit: 10 Jun 2025
Posts: 183
Own Kudos:
Given Kudos: 93
Location: India
Posts: 183
Kudos: 23
Kudos
Add Kudos
Bookmarks
Bookmark this Post
VeritasKarishma

if in option b All is mentioned instead of some will then optin is correct

like in this question it's not 100 % full proof weaken ans
i have seen many questions like the same which aren't 100% proof ans in strenthen and weaken type. So should i look for options which just provide belief of either strengthening or weakening

Any other insights you wants to give regarding this type of answers
User avatar
KarishmaB
Joined: 16 Oct 2010
Last visit: 18 Nov 2025
Posts: 16,267
Own Kudos:
76,984
 [1]
Given Kudos: 482
Location: Pune, India
Expert
Expert reply
Active GMAT Club Expert! Tag them with @ followed by their username for a faster response.
Posts: 16,267
Kudos: 76,984
 [1]
1
Kudos
Add Kudos
Bookmarks
Bookmark this Post
saby1410
VeritasKarishma

if in option b All is mentioned instead of some will then optin is correct

like in this question it's not 100 % full proof weaken ans
i have seen many questions like the same which aren't 100% proof ans in strenthen and weaken type. So should i look for options which just provide belief of either strengthening or weakening

Any other insights you wants to give regarding this type of answers

If (B) has "all" in place of "some", it will negate this premise:
"studies have shown that the older people get, the less likely they are to eat in fast-food restaurants"

We need to take the premises to be true. It doesn't make sense then to have its negation true too.
User avatar
realslimsiddy
Joined: 18 Aug 2019
Last visit: 11 Dec 2020
Posts: 10
Own Kudos:
Given Kudos: 8
Posts: 10
Kudos: 2
Kudos
Add Kudos
Bookmarks
Bookmark this Post
I rejected option D on the grounds that "population increase" could also be due to people moving to this city and there could be a possibility that majority of these people moving into the city could be old people(?) Due to this ambiguity I Went for option B which i wasn't also a fan of. Can someone tell me why it would be incorrect to think like this?
User avatar
GMATNinja
User avatar
GMAT Club Verbal Expert
Joined: 13 Aug 2009
Last visit: 18 Nov 2025
Posts: 7,445
Own Kudos:
Given Kudos: 2,060
Status: GMAT/GRE/LSAT tutors
Location: United States (CO)
GMAT 1: 780 Q51 V46
GMAT 2: 800 Q51 V51
GRE 1: Q170 V170
GRE 2: Q170 V170
Products:
Expert
Expert reply
GMAT 2: 800 Q51 V51
GRE 1: Q170 V170
GRE 2: Q170 V170
Posts: 7,445
Kudos: 69,780
Kudos
Add Kudos
Bookmarks
Bookmark this Post
realslimsiddy
I rejected option D on the grounds that "population increase" could also be due to people moving to this city and there could be a possibility that majority of these people moving into the city could be old people(?) Due to this ambiguity I Went for option B which i wasn't also a fan of. Can someone tell me why it would be incorrect to think like this?
We posted an explanation of why answer choice (B) is incorrect here and an explanation of why answer choice (D) is correct here. Let us know if those don't clear things up!
User avatar
jabhatta2
Joined: 15 Dec 2016
Last visit: 21 Apr 2023
Posts: 1,294
Own Kudos:
Given Kudos: 188
Posts: 1,294
Kudos: 317
Kudos
Add Kudos
Bookmarks
Bookmark this Post
GMATNinja
scarlatti
Vyshak
Younger people eat in fastfood. Average age of current population is increasing - Number of fastfood restaurants will decrease

Possible weakener: As the current population ages, younger people are added to the population

A) Fast-food restaurants in Canatria are getting bigger, so each one can serve more customers. - Incorrect - Out of context. We are not bothered about the increase in capacity of fast food restaurants.

B) Some older people eat at fast-food restaurants more frequently than the average young person. - Incorrect - Irrelevant

C) Many people who rarely eat in fast-food restaurants nevertheless eat regularly in restaurants. - Incorrect - Strengthens

D) The overall population of Canatria is growing steadily. - Correct

E) As the population of Canatria gets older, more people are eating at home. - Incorrect - Supports the argument

Answer: D

I have a problem with D. Just because Canatria´s population increased, it doesn´t mean that the number of young people in Canatria will increase, since that same increase in the population could have been due to immigration for example, and we can´t say that immigrants are young.
Vyshak, it's true that choice (D) might not weaken the argument. We can think of scenarios, like the one you described, in which choice (D) would have no effect. But remember that we are looking for an answer choice that most seriously weakens the argument, not an answer choice that definitely weakens the argument.

The author says that the number of fast-food restaurants is likely to decrease because the average age is increasing. Thus, according to the author, the number of young people will probably decrease. Choice (D) seriously undermines this reasoning. If the overall population is growing steadily ("steadily" is a key word here), then that will probably counteract the effect of the aging.

Sure, we might have a steady influx of older immigrants, but if we are talking about steady growth of the overall population, it is more likely that we will see increases across all ages. Choice (D) does not disprove the author's logic with 100% certainty, but it does seriously weaken the argument.

I hope that helps!

Hi GMATNinja - wanted to go over your statement in the yellow highlight. Mentioned in the yellow with regards to D -- there are some scenario that weaken and some scenario's that don't weaken when it comes to option D

Hence option D should not automatically be eliminated.

But according to this logic, even then there are scenario's in option B which weaken

--In option B, word -- the word "Some" can mean everything from 1 % to 30 % to 49 % to all 100 % of the people. That is the clinical definition of the word "some" (anything but zero)

So option B -- there are scenario's where perhaps option B is referring to 20 % or 30 % or 49 % of the elderly that order more frequently that young folks (it cannot be 50 % or more as that would be go against the premise)

Hence in this specific scenario's (20 % or 30 % or 49 %) option B -- you would agree , option B would also weaken the argument by a little bit.

Just wondering what your thoughts are on something like this !

Thank you for everything !
User avatar
GMATNinja
User avatar
GMAT Club Verbal Expert
Joined: 13 Aug 2009
Last visit: 18 Nov 2025
Posts: 7,445
Own Kudos:
69,780
 [2]
Given Kudos: 2,060
Status: GMAT/GRE/LSAT tutors
Location: United States (CO)
GMAT 1: 780 Q51 V46
GMAT 2: 800 Q51 V51
GRE 1: Q170 V170
GRE 2: Q170 V170
Products:
Expert
Expert reply
GMAT 2: 800 Q51 V51
GRE 1: Q170 V170
GRE 2: Q170 V170
Posts: 7,445
Kudos: 69,780
 [2]
2
Kudos
Add Kudos
Bookmarks
Bookmark this Post
jabhatta2
GMATNinja
scarlatti
I have a problem with D. Just because Canatria´s population increased, it doesn´t mean that the number of young people in Canatria will increase, since that same increase in the population could have been due to immigration for example, and we can´t say that immigrants are young.
Vyshak, it's true that choice (D) might not weaken the argument. We can think of scenarios, like the one you described, in which choice (D) would have no effect. But remember that we are looking for an answer choice that most seriously weakens the argument, not an answer choice that definitely weakens the argument.

The author says that the number of fast-food restaurants is likely to decrease because the average age is increasing. Thus, according to the author, the number of young people will probably decrease. Choice (D) seriously undermines this reasoning. If the overall population is growing steadily ("steadily" is a key word here), then that will probably counteract the effect of the aging.

Sure, we might have a steady influx of older immigrants, but if we are talking about steady growth of the overall population, it is more likely that we will see increases across all ages. Choice (D) does not disprove the author's logic with 100% certainty, but it does seriously weaken the argument.

I hope that helps!

Hi GMATNinja - wanted to go over your statement in the yellow highlight. Mentioned in the yellow with regards to D -- there are some scenario that weaken and some scenario's that don't weaken when it comes to option D

Hence option D should not automatically be eliminated.

But according to this logic, even then there are scenario's in option B which weaken

--In option B, word -- the word "Some" can mean everything from 1 % to 30 % to 49 % to all 100 % of the people. That is the clinical definition of the word "some" (anything but zero)

So option B -- there are scenario's where perhaps option B is referring to 20 % or 30 % or 49 % of the elderly that order more frequently that young folks (it cannot be 50 % or more as that would be go against the premise)

Hence in this specific scenario's (20 % or 30 % or 49 %) option B -- you would agree , option B would also weaken the argument by a little bit.

Just wondering what your thoughts are on something like this !

Thank you for everything !

Quote:
Fast-food restaurants make up 45 percent of all restaurants in Cantaria. Customers at these restaurants tend to be young; in fact, studies have shown that the older people get, the less likely they are to eat in fast-food restaurants. Since the average age of the Canatrian population is gradually rising and will continue to do so, the number of fast-food restaurants is likely to decrease.

Which of the following, if true, most seriously weakens the argument?

(B) Some older people eat at fast-food restaurants more frequently than the average young person.
Great question! Even if it's true that some older people eat at fast-food restaurants more than the average young person does, this doesn't affect the fact given in the argument that the older you get, the less fast food you eat. So in the future, as the population gets older, people will eat less fast-food -- even if there are some old people who still eat more fast food than all the teenagers.

If you want to think about it a bit more quantitatively, the argument is telling us that there's a distribution for young people and another distribution for old people and that the mean for old people is lower than the mean for young people. Answer choice (B) is telling us that the standard deviation for old people is high because there are some old people who eat more than the average young person (and therefore a whole lot more than the average old person). But if we're concerned with the total demand for fast-food, we only care about the mean for each distribution and how many people are in each group (because mean consumption will equal total consumption divided by n). The standard deviation doesn't matter.

I hope this helps!
User avatar
jabhatta2
Joined: 15 Dec 2016
Last visit: 21 Apr 2023
Posts: 1,294
Own Kudos:
Given Kudos: 188
Posts: 1,294
Kudos: 317
Kudos
Add Kudos
Bookmarks
Bookmark this Post
VeritasKarishma
saby1410
VeritasKarishma

if in option b All is mentioned instead of some will then optin is correct

like in this question it's not 100 % full proof weaken ans
i have seen many questions like the same which aren't 100% proof ans in strenthen and weaken type. So should i look for options which just provide belief of either strengthening or weakening

Any other insights you wants to give regarding this type of answers

If (B) has "all" in place of "some", it will negate this premise:
"studies have shown that the older people get, the less likely they are to eat in fast-food restaurants"

We need to take the premises to be true. It doesn't make sense then to have its negation true too.

Hi VeritasKarishma - what if option B had "49 % of the older people eat at fast food restaurants more frequently than the average young person"

Would that weaken in your view ?

Initially I thought it would weaken but now I am thinking , even if the percentage is 49 % or 100 %, it may not go against the premise

""studies have shown that the older people get, the less likely they are to eat in fast-food restaurants"

because it quite possible that 49 % or 100 % of the elderly just frequent fast food restaurants and it's quite possible they just order a coffee (1 $ value) whereas the young may frequent less and order on average 15 $ worth of food perhaps

Thoughts on if (B) was 100 % or 49 % instead of 'Some" -- would that still not weaken or would that weaken ?

Thank you !

Thank you !
User avatar
KarishmaB
Joined: 16 Oct 2010
Last visit: 18 Nov 2025
Posts: 16,267
Own Kudos:
76,984
 [2]
Given Kudos: 482
Location: Pune, India
Expert
Expert reply
Active GMAT Club Expert! Tag them with @ followed by their username for a faster response.
Posts: 16,267
Kudos: 76,984
 [2]
2
Kudos
Add Kudos
Bookmarks
Bookmark this Post
jabhatta2
VeritasKarishma
saby1410
VeritasKarishma

if in option b All is mentioned instead of some will then optin is correct

like in this question it's not 100 % full proof weaken ans
i have seen many questions like the same which aren't 100% proof ans in strenthen and weaken type. So should i look for options which just provide belief of either strengthening or weakening

Any other insights you wants to give regarding this type of answers

If (B) has "all" in place of "some", it will negate this premise:
"studies have shown that the older people get, the less likely they are to eat in fast-food restaurants"

We need to take the premises to be true. It doesn't make sense then to have its negation true too.

Hi VeritasKarishma - what if option B had "49 % of the older people eat at fast food restaurants more frequently than the average young person"

Would that weaken in your view ?

Initially I thought it would weaken but now I am thinking , even if the percentage is 49 % or 100 %, it may not go against the premise

""studies have shown that the older people get, the less likely they are to eat in fast-food restaurants"

because it quite possible that 49 % or 100 % of the elderly just frequent fast food restaurants and it's quite possible they just order a coffee (1 $ value) whereas the young may frequent less and order on average 15 $ worth of food perhaps

Thoughts on if (B) was 100 % or 49 % instead of 'Some" -- would that still not weaken or would that weaken ?

Thank you !

Thank you !

Given premise:
"studies have shown that the older people get, the less likely they are to eat in fast-food restaurants. "

So people at 40 yrs of age eat less frequently in fast food restaurants than do people at 20 yrs of age. Either a smaller fraction of 40 yr olds may be eating in these restaurants or all may be eating but less frequently than 20 yr olds.
Even if 100% 40 yr olds eat at fast food restaurants, they may be doing so only once a month while 20 yr olds may be eating there every week.

Option (B) says that "some" older people eat more frequently than do youngsters so they are giving "some" as the exception group to the rule that older people are less likely to eat. Had this been "all older people eat more frequently than do youngsters," that would make the premise false. So an option like that would be unacceptable.

The distinction is not between 'eat' and 'visit'. The argument as well as the option (B) talks about 'eat'.
 1   2   
Moderators:
GMAT Club Verbal Expert
7445 posts
GMAT Club Verbal Expert
234 posts
188 posts